LSAT and Law School Admissions Forum

Get expert LSAT preparation and law school admissions advice from PowerScore Test Preparation.

User avatar
 Dave Killoran
PowerScore Staff
  • PowerScore Staff
  • Posts: 5850
  • Joined: Mar 25, 2011
|
#41286
Complete Question Explanation
(The complete setup for this game can be found here: lsat/viewtopic.php?t=15690)

The correct answer choice is (A)

If K delivers the third speech, there is no immediate inference that follows. The question stem asks for which variable cannot give the fourth speech, and the rule that seems most likely to come into play is the first rule about H, J, and K.

Answer choice (A) is correct because if K is third and H is fourth, then J must be second or fifth (because J cannot give the first or last speech). Thus, no matter where J is placed, there is a violation of the first rule.
 medialaw111516
  • Posts: 80
  • Joined: Dec 11, 2018
|
#71082
So I picked A by making a diagram and just trying out all of the answer choices like I always do with the type of question (luckily A was correct and I didn't have to do another one). Is there a fast way to do questions like this, or should I just be trying all the answers?
 Jeremy Press
PowerScore Staff
  • PowerScore Staff
  • Posts: 1000
  • Joined: Jun 12, 2017
|
#71108
Hi medialaw,

Since this is a cannot be true question, you're looking for an answer that's likely to restrict the game in ways that make a solution impossible. Since the local condition involves K, as the original post notes, you can expect that the "relevant rule" that's likely to come into play here is the rule about H, J, and K. With that in mind, it makes sense to at least begin the testing process with answer choices A and B, because placing a second variable from the H, J, K rule restricts the solution quite heavily. That gives you a sensible way of narrowing your testing process to two likely answers that will probably eliminate the need to test all five answers. But, if you want to go a step further than that, J is the most restricted variable of the H, J, K trio (because of the rule that it can't go first or last). So, what's most likely to create problems is forcing H and K into positions that leave you with that already very restricted variable to place. So, answer choice A turns out to be the best starting point for testing purposes.

I hope this helps!

Jeremy

Get the most out of your LSAT Prep Plus subscription.

Analyze and track your performance with our Testing and Analytics Package.